18. Sequences

\(\displaystyle \lim_{n\to\infty}a_n=L\)   means For all   \(\varepsilon \gt 0\),   there is a positive integer,   \(N\),   such that if   \(n \gt N\)   then   \(|a_n-L| \lt \varepsilon\). \(\displaystyle \lim_{n\to\infty}a_n=\infty\)   means For all   \(M \gt 0\),   there is a positive integer,   \(N\),   such that if   \(n \gt N\)   then   \(a_n \gt M\).

e4. Precise Bounded Monotonic Sequence Theorem


1) Every increasing sequence which is bounded above is convergent to the least upper bound.
2) Every decreasing sequence which is bounded below is convergent to the greatest lower bound.

3) Every bounded, monotonic sequence is convergent.

4) Every unbounded, increasing sequence is divergent to \(\infty\).
5) Every unbounded, decreasing sequence is divergent to \(-\infty\).

Proof of (1):
Let \(a_n\) be an increasing sequence which is bounded above and let \(L\) be its least upper bound. Then we need to show \(\displaystyle \lim_{n\to\infty}a_n=L\) which means: For all   \(\varepsilon \gt 0\),   there is a positive integer,   \(N\),   such that if   \(n \gt N\)   then   \(|a_n-L| \lt \varepsilon\). So given an arbitrary number \(\varepsilon \gt 0\), we claim there is an index \(N\) with \(a_N \gt L-\varepsilon\). If not, then every \(a_n\) satisfies \(a_n \lt L-\varepsilon\) and \(L-\varepsilon\) would be an upper bound. But this is impossible because \(L\) is the least upper bound. Consequently, there is an \(N\) for which \(a_N \gt L-\varepsilon\). Now suppose \(n \gt N\). Then \(a_n \gt a_N\) because the sequence is increasing. So: \[ a_n \gt a_N \gt L-\varepsilon \quad \text{or} \quad \varepsilon \gt L-a_n=|a_n-L| \]

Proof of (2):
The proof is totally analogous to that for (1), reversing some signs and some inequalities.

Proof of (3):
If the sequence if bounded and monotonic, then it is either increasing and bounded above or decreasing and bounded below. In either case it converges.

Proof of (4):
Let \(a_n\) be an increasing sequence which is unbounded. We need to show \(\displaystyle \lim_{n\to\infty}a_n=\infty\) which means: For all   \(M \gt 0\),   there is a positive integer,   \(N\),   such that if   \(n \gt N\)   then   \(a_n \gt M\). Since the sequence \(a_n\) is increasing it is bounded below by \(a_1\). Since it is unbounded, it must be unbounded above. That means it gets bigger than any given number. In particular, given an arbitrary number \(M \gt 0\), there is a number \(N\) for which \(a_N \gt M\). Then if \(n \gt N\) then \(a_n \gt a_N\) because the sequence is increasing and so \(a_n \gt a_N \gt M\).

Proof of (5):
The proof is totally analogous to that for (4), reversing some signs and some inequalities.

© MYMathApps

Supported in part by NSF Grant #1123255